Showing the basis of a Hilbert Space have the same cardinality The 2019 Stack Overflow Developer Survey Results Are In Announcing the arrival of Valued Associate #679: Cesar Manara Planned maintenance scheduled April 17/18, 2019 at 00:00UTC (8:00pm US/Eastern)Uncountable basis and separabilitysynthesis operator bijective $Rightarrow exists$ ONB $(psi_i)_i in I$, $Q in L(mathcal H)$ bijective: $Qpsi_i = varphi_i$Orthonormal basis in Hilbert space - 2 questionsCan we have infinite-dimensional separable Hilbert spaces?If $B$ spans a dense subspace of a Hilbert space $H$, is it a basis for $H$?Countable basis representation in Hilbert spaceSimple question about basis of Hilbert spaceSchauder basis that is not Hilbert basis“Basis” of non-separable Hilbert spaceBasis of infinite dimensional Banach space and separable hilbert spaceOrthonormal basis in every Hilbert space?$H$ is a Hilbert space , then each orthonormal basis of $H$ has the same cardinality

Do warforged have souls?

system() function string length limit

Is it ok to offer lower paid work as a trial period before negotiating for a full-time job?

He got a vote 80% that of Emmanuel Macron’s

Finding the path in a graph from A to B then back to A with a minimum of shared edges

Why can't wing-mounted spoilers be used to steepen approaches?

Can undead you have reanimated wait inside a portable hole?

How to delete random line from file using Unix command?

Working through the single responsibility principle (SRP) in Python when calls are expensive

Would an alien lifeform be able to achieve space travel if lacking in vision?

What is special about square numbers here?

How do I add random spotting to the same face in cycles?

Single author papers against my advisor's will?

What's the point in a preamp?

Who or what is the being for whom Being is a question for Heidegger?

Did the UK government pay "millions and millions of dollars" to try to snag Julian Assange?

Problems with Ubuntu mount /tmp

Road tyres vs "Street" tyres for charity ride on MTB Tandem

ELI5: Why do they say that Israel would have been the fourth country to land a spacecraft on the Moon and why do they call it low cost?

Did the new image of black hole confirm the general theory of relativity?

Is every episode of "Where are my Pants?" identical?

The variadic template constructor of my class cannot modify my class members, why is that so?

What information about me do stores get via my credit card?

Do working physicists consider Newtonian mechanics to be "falsified"?



Showing the basis of a Hilbert Space have the same cardinality



The 2019 Stack Overflow Developer Survey Results Are In
Announcing the arrival of Valued Associate #679: Cesar Manara
Planned maintenance scheduled April 17/18, 2019 at 00:00UTC (8:00pm US/Eastern)Uncountable basis and separabilitysynthesis operator bijective $Rightarrow exists$ ONB $(psi_i)_i in I$, $Q in L(mathcal H)$ bijective: $Qpsi_i = varphi_i$Orthonormal basis in Hilbert space - 2 questionsCan we have infinite-dimensional separable Hilbert spaces?If $B$ spans a dense subspace of a Hilbert space $H$, is it a basis for $H$?Countable basis representation in Hilbert spaceSimple question about basis of Hilbert spaceSchauder basis that is not Hilbert basis“Basis” of non-separable Hilbert spaceBasis of infinite dimensional Banach space and separable hilbert spaceOrthonormal basis in every Hilbert space?$H$ is a Hilbert space , then each orthonormal basis of $H$ has the same cardinality










12












$begingroup$


I am trying to show that if we have two orthonormal families $a_i_iin K$ and $b_j_jin S$ and these are the basis of some Hilbert Space H, then they have the same cardinality.



So If I suppose that the $a_i_iin K$ is countable, i.e., that $K$ is countable and that $S$ is uncountable, then we want to show that this leads to a contradiction.



As the $b_i_iin K$ forms a basis we know that and $a_n$ as:



$$a_n=sum_1^infty c_ib_i$$



Now if we let the set $D_n=i$ and then take:



$D:=bigcup_n^infty D_n$ then this is the set of all indices and it is a countable set as it is the countable union of countable sets.



Take $linS$ that is not in $D$ then as $b_lin H$ and as the $a_i_iin K$ forms a basis we have that:



$$b_lin overlinelinsum_i=1^infty a_i$$



Then from above we have that:



$$overlinelinsum_i=1^infty a_i=overlinelinb_d$$



If we now consider $$ ||b_l||^2=sum _1^infty c_ilangle b_d_i, b_l rangle =0$$ so we have the contradiction.



So is the above proof correct and can we generalise this further to different cardinalities? Does H have to be a Hilbert space for this to be true?



Thanks very much for any help.










share|cite|improve this question











$endgroup$







  • 2




    $begingroup$
    Seems right. But it is hardly readable, for you are mixing $D$, $u$, $mathbb S$, $b$ up.
    $endgroup$
    – martini
    Nov 7 '12 at 14:12











  • $begingroup$
    @martini oh dear, just looked it over, I stopped halfway through and then continued on and used different notation by mistake, sorry . thanks for the comment
    $endgroup$
    – hmmmm
    Nov 7 '12 at 14:16






  • 2




    $begingroup$
    See Halmos, Introduction to Hilbert Space and the theory of spectral multiplicity, §16 Dimension, Theorem 1.
    $endgroup$
    – vesszabo
    Nov 7 '12 at 20:16















12












$begingroup$


I am trying to show that if we have two orthonormal families $a_i_iin K$ and $b_j_jin S$ and these are the basis of some Hilbert Space H, then they have the same cardinality.



So If I suppose that the $a_i_iin K$ is countable, i.e., that $K$ is countable and that $S$ is uncountable, then we want to show that this leads to a contradiction.



As the $b_i_iin K$ forms a basis we know that and $a_n$ as:



$$a_n=sum_1^infty c_ib_i$$



Now if we let the set $D_n=i$ and then take:



$D:=bigcup_n^infty D_n$ then this is the set of all indices and it is a countable set as it is the countable union of countable sets.



Take $linS$ that is not in $D$ then as $b_lin H$ and as the $a_i_iin K$ forms a basis we have that:



$$b_lin overlinelinsum_i=1^infty a_i$$



Then from above we have that:



$$overlinelinsum_i=1^infty a_i=overlinelinb_d$$



If we now consider $$ ||b_l||^2=sum _1^infty c_ilangle b_d_i, b_l rangle =0$$ so we have the contradiction.



So is the above proof correct and can we generalise this further to different cardinalities? Does H have to be a Hilbert space for this to be true?



Thanks very much for any help.










share|cite|improve this question











$endgroup$







  • 2




    $begingroup$
    Seems right. But it is hardly readable, for you are mixing $D$, $u$, $mathbb S$, $b$ up.
    $endgroup$
    – martini
    Nov 7 '12 at 14:12











  • $begingroup$
    @martini oh dear, just looked it over, I stopped halfway through and then continued on and used different notation by mistake, sorry . thanks for the comment
    $endgroup$
    – hmmmm
    Nov 7 '12 at 14:16






  • 2




    $begingroup$
    See Halmos, Introduction to Hilbert Space and the theory of spectral multiplicity, §16 Dimension, Theorem 1.
    $endgroup$
    – vesszabo
    Nov 7 '12 at 20:16













12












12








12


7



$begingroup$


I am trying to show that if we have two orthonormal families $a_i_iin K$ and $b_j_jin S$ and these are the basis of some Hilbert Space H, then they have the same cardinality.



So If I suppose that the $a_i_iin K$ is countable, i.e., that $K$ is countable and that $S$ is uncountable, then we want to show that this leads to a contradiction.



As the $b_i_iin K$ forms a basis we know that and $a_n$ as:



$$a_n=sum_1^infty c_ib_i$$



Now if we let the set $D_n=i$ and then take:



$D:=bigcup_n^infty D_n$ then this is the set of all indices and it is a countable set as it is the countable union of countable sets.



Take $linS$ that is not in $D$ then as $b_lin H$ and as the $a_i_iin K$ forms a basis we have that:



$$b_lin overlinelinsum_i=1^infty a_i$$



Then from above we have that:



$$overlinelinsum_i=1^infty a_i=overlinelinb_d$$



If we now consider $$ ||b_l||^2=sum _1^infty c_ilangle b_d_i, b_l rangle =0$$ so we have the contradiction.



So is the above proof correct and can we generalise this further to different cardinalities? Does H have to be a Hilbert space for this to be true?



Thanks very much for any help.










share|cite|improve this question











$endgroup$




I am trying to show that if we have two orthonormal families $a_i_iin K$ and $b_j_jin S$ and these are the basis of some Hilbert Space H, then they have the same cardinality.



So If I suppose that the $a_i_iin K$ is countable, i.e., that $K$ is countable and that $S$ is uncountable, then we want to show that this leads to a contradiction.



As the $b_i_iin K$ forms a basis we know that and $a_n$ as:



$$a_n=sum_1^infty c_ib_i$$



Now if we let the set $D_n=i$ and then take:



$D:=bigcup_n^infty D_n$ then this is the set of all indices and it is a countable set as it is the countable union of countable sets.



Take $linS$ that is not in $D$ then as $b_lin H$ and as the $a_i_iin K$ forms a basis we have that:



$$b_lin overlinelinsum_i=1^infty a_i$$



Then from above we have that:



$$overlinelinsum_i=1^infty a_i=overlinelinb_d$$



If we now consider $$ ||b_l||^2=sum _1^infty c_ilangle b_d_i, b_l rangle =0$$ so we have the contradiction.



So is the above proof correct and can we generalise this further to different cardinalities? Does H have to be a Hilbert space for this to be true?



Thanks very much for any help.







hilbert-spaces






share|cite|improve this question















share|cite|improve this question













share|cite|improve this question




share|cite|improve this question








edited Jun 21 '16 at 19:27









Watson

16k92971




16k92971










asked Nov 7 '12 at 14:03









hmmmmhmmmm

2,68422861




2,68422861







  • 2




    $begingroup$
    Seems right. But it is hardly readable, for you are mixing $D$, $u$, $mathbb S$, $b$ up.
    $endgroup$
    – martini
    Nov 7 '12 at 14:12











  • $begingroup$
    @martini oh dear, just looked it over, I stopped halfway through and then continued on and used different notation by mistake, sorry . thanks for the comment
    $endgroup$
    – hmmmm
    Nov 7 '12 at 14:16






  • 2




    $begingroup$
    See Halmos, Introduction to Hilbert Space and the theory of spectral multiplicity, §16 Dimension, Theorem 1.
    $endgroup$
    – vesszabo
    Nov 7 '12 at 20:16












  • 2




    $begingroup$
    Seems right. But it is hardly readable, for you are mixing $D$, $u$, $mathbb S$, $b$ up.
    $endgroup$
    – martini
    Nov 7 '12 at 14:12











  • $begingroup$
    @martini oh dear, just looked it over, I stopped halfway through and then continued on and used different notation by mistake, sorry . thanks for the comment
    $endgroup$
    – hmmmm
    Nov 7 '12 at 14:16






  • 2




    $begingroup$
    See Halmos, Introduction to Hilbert Space and the theory of spectral multiplicity, §16 Dimension, Theorem 1.
    $endgroup$
    – vesszabo
    Nov 7 '12 at 20:16







2




2




$begingroup$
Seems right. But it is hardly readable, for you are mixing $D$, $u$, $mathbb S$, $b$ up.
$endgroup$
– martini
Nov 7 '12 at 14:12





$begingroup$
Seems right. But it is hardly readable, for you are mixing $D$, $u$, $mathbb S$, $b$ up.
$endgroup$
– martini
Nov 7 '12 at 14:12













$begingroup$
@martini oh dear, just looked it over, I stopped halfway through and then continued on and used different notation by mistake, sorry . thanks for the comment
$endgroup$
– hmmmm
Nov 7 '12 at 14:16




$begingroup$
@martini oh dear, just looked it over, I stopped halfway through and then continued on and used different notation by mistake, sorry . thanks for the comment
$endgroup$
– hmmmm
Nov 7 '12 at 14:16




2




2




$begingroup$
See Halmos, Introduction to Hilbert Space and the theory of spectral multiplicity, §16 Dimension, Theorem 1.
$endgroup$
– vesszabo
Nov 7 '12 at 20:16




$begingroup$
See Halmos, Introduction to Hilbert Space and the theory of spectral multiplicity, §16 Dimension, Theorem 1.
$endgroup$
– vesszabo
Nov 7 '12 at 20:16










1 Answer
1






active

oldest

votes


















11












$begingroup$

Let $(a_i)_i in K$ and $(b_j)_jin S$ two bases of a Hilbert space $H$. Following your idea, we will show that $|S| le |K|$ and by symmetry conclude $|S| = |K|$. Note that for finite dimensional spaces everything we are up to is well known from basic linear algebra. So we will suppose $aleph_0 le |K|, |S|$ in what follows.



Let $i in K$, then we can write, as the $(b_j)$ form a basis
[ a_i = sum_jin S_i langle a_i, b_jrangle b_j ]
for some countable subset $S_i subseteq S$. Let $S' := bigcup_iin K S_i$. If there were any $l in S setminus S'$, we have, as $(a_i)$ is a basis,
$$ b_l in H = overlineoperatornamelina_i : i in K $$
on the other hand, for each $i$
$$ a_i in overlineoperatornamelinb_j: j in S_i subseteq
overlineoperatornamelinb_j : j in S' $$

so $b_l in overlineoperatornamelinb_j : j in S'$, hence there is a countable $T subseteq S'$ with $b_l in overlineoperatornamelinb_j : j in T$, giving
$$ |b_l|^2 = sum_j in T|langle b_l, b_jrangle|^2 = 0 $$
Contradiction.



So $S = S'$ and hence
$$ |S| = |S'| = left|bigcup_i in K S_iright| le |K| cdotsup_i |S_i| le |K| cdot aleph_0 = |K| $$
as desired.






share|cite|improve this answer











$endgroup$













    Your Answer








    StackExchange.ready(function()
    var channelOptions =
    tags: "".split(" "),
    id: "69"
    ;
    initTagRenderer("".split(" "), "".split(" "), channelOptions);

    StackExchange.using("externalEditor", function()
    // Have to fire editor after snippets, if snippets enabled
    if (StackExchange.settings.snippets.snippetsEnabled)
    StackExchange.using("snippets", function()
    createEditor();
    );

    else
    createEditor();

    );

    function createEditor()
    StackExchange.prepareEditor(
    heartbeatType: 'answer',
    autoActivateHeartbeat: false,
    convertImagesToLinks: true,
    noModals: true,
    showLowRepImageUploadWarning: true,
    reputationToPostImages: 10,
    bindNavPrevention: true,
    postfix: "",
    imageUploader:
    brandingHtml: "Powered by u003ca class="icon-imgur-white" href="https://imgur.com/"u003eu003c/au003e",
    contentPolicyHtml: "User contributions licensed under u003ca href="https://creativecommons.org/licenses/by-sa/3.0/"u003ecc by-sa 3.0 with attribution requiredu003c/au003e u003ca href="https://stackoverflow.com/legal/content-policy"u003e(content policy)u003c/au003e",
    allowUrls: true
    ,
    noCode: true, onDemand: true,
    discardSelector: ".discard-answer"
    ,immediatelyShowMarkdownHelp:true
    );



    );













    draft saved

    draft discarded


















    StackExchange.ready(
    function ()
    StackExchange.openid.initPostLogin('.new-post-login', 'https%3a%2f%2fmath.stackexchange.com%2fquestions%2f232166%2fshowing-the-basis-of-a-hilbert-space-have-the-same-cardinality%23new-answer', 'question_page');

    );

    Post as a guest















    Required, but never shown

























    1 Answer
    1






    active

    oldest

    votes








    1 Answer
    1






    active

    oldest

    votes









    active

    oldest

    votes






    active

    oldest

    votes









    11












    $begingroup$

    Let $(a_i)_i in K$ and $(b_j)_jin S$ two bases of a Hilbert space $H$. Following your idea, we will show that $|S| le |K|$ and by symmetry conclude $|S| = |K|$. Note that for finite dimensional spaces everything we are up to is well known from basic linear algebra. So we will suppose $aleph_0 le |K|, |S|$ in what follows.



    Let $i in K$, then we can write, as the $(b_j)$ form a basis
    [ a_i = sum_jin S_i langle a_i, b_jrangle b_j ]
    for some countable subset $S_i subseteq S$. Let $S' := bigcup_iin K S_i$. If there were any $l in S setminus S'$, we have, as $(a_i)$ is a basis,
    $$ b_l in H = overlineoperatornamelina_i : i in K $$
    on the other hand, for each $i$
    $$ a_i in overlineoperatornamelinb_j: j in S_i subseteq
    overlineoperatornamelinb_j : j in S' $$

    so $b_l in overlineoperatornamelinb_j : j in S'$, hence there is a countable $T subseteq S'$ with $b_l in overlineoperatornamelinb_j : j in T$, giving
    $$ |b_l|^2 = sum_j in T|langle b_l, b_jrangle|^2 = 0 $$
    Contradiction.



    So $S = S'$ and hence
    $$ |S| = |S'| = left|bigcup_i in K S_iright| le |K| cdotsup_i |S_i| le |K| cdot aleph_0 = |K| $$
    as desired.






    share|cite|improve this answer











    $endgroup$

















      11












      $begingroup$

      Let $(a_i)_i in K$ and $(b_j)_jin S$ two bases of a Hilbert space $H$. Following your idea, we will show that $|S| le |K|$ and by symmetry conclude $|S| = |K|$. Note that for finite dimensional spaces everything we are up to is well known from basic linear algebra. So we will suppose $aleph_0 le |K|, |S|$ in what follows.



      Let $i in K$, then we can write, as the $(b_j)$ form a basis
      [ a_i = sum_jin S_i langle a_i, b_jrangle b_j ]
      for some countable subset $S_i subseteq S$. Let $S' := bigcup_iin K S_i$. If there were any $l in S setminus S'$, we have, as $(a_i)$ is a basis,
      $$ b_l in H = overlineoperatornamelina_i : i in K $$
      on the other hand, for each $i$
      $$ a_i in overlineoperatornamelinb_j: j in S_i subseteq
      overlineoperatornamelinb_j : j in S' $$

      so $b_l in overlineoperatornamelinb_j : j in S'$, hence there is a countable $T subseteq S'$ with $b_l in overlineoperatornamelinb_j : j in T$, giving
      $$ |b_l|^2 = sum_j in T|langle b_l, b_jrangle|^2 = 0 $$
      Contradiction.



      So $S = S'$ and hence
      $$ |S| = |S'| = left|bigcup_i in K S_iright| le |K| cdotsup_i |S_i| le |K| cdot aleph_0 = |K| $$
      as desired.






      share|cite|improve this answer











      $endgroup$















        11












        11








        11





        $begingroup$

        Let $(a_i)_i in K$ and $(b_j)_jin S$ two bases of a Hilbert space $H$. Following your idea, we will show that $|S| le |K|$ and by symmetry conclude $|S| = |K|$. Note that for finite dimensional spaces everything we are up to is well known from basic linear algebra. So we will suppose $aleph_0 le |K|, |S|$ in what follows.



        Let $i in K$, then we can write, as the $(b_j)$ form a basis
        [ a_i = sum_jin S_i langle a_i, b_jrangle b_j ]
        for some countable subset $S_i subseteq S$. Let $S' := bigcup_iin K S_i$. If there were any $l in S setminus S'$, we have, as $(a_i)$ is a basis,
        $$ b_l in H = overlineoperatornamelina_i : i in K $$
        on the other hand, for each $i$
        $$ a_i in overlineoperatornamelinb_j: j in S_i subseteq
        overlineoperatornamelinb_j : j in S' $$

        so $b_l in overlineoperatornamelinb_j : j in S'$, hence there is a countable $T subseteq S'$ with $b_l in overlineoperatornamelinb_j : j in T$, giving
        $$ |b_l|^2 = sum_j in T|langle b_l, b_jrangle|^2 = 0 $$
        Contradiction.



        So $S = S'$ and hence
        $$ |S| = |S'| = left|bigcup_i in K S_iright| le |K| cdotsup_i |S_i| le |K| cdot aleph_0 = |K| $$
        as desired.






        share|cite|improve this answer











        $endgroup$



        Let $(a_i)_i in K$ and $(b_j)_jin S$ two bases of a Hilbert space $H$. Following your idea, we will show that $|S| le |K|$ and by symmetry conclude $|S| = |K|$. Note that for finite dimensional spaces everything we are up to is well known from basic linear algebra. So we will suppose $aleph_0 le |K|, |S|$ in what follows.



        Let $i in K$, then we can write, as the $(b_j)$ form a basis
        [ a_i = sum_jin S_i langle a_i, b_jrangle b_j ]
        for some countable subset $S_i subseteq S$. Let $S' := bigcup_iin K S_i$. If there were any $l in S setminus S'$, we have, as $(a_i)$ is a basis,
        $$ b_l in H = overlineoperatornamelina_i : i in K $$
        on the other hand, for each $i$
        $$ a_i in overlineoperatornamelinb_j: j in S_i subseteq
        overlineoperatornamelinb_j : j in S' $$

        so $b_l in overlineoperatornamelinb_j : j in S'$, hence there is a countable $T subseteq S'$ with $b_l in overlineoperatornamelinb_j : j in T$, giving
        $$ |b_l|^2 = sum_j in T|langle b_l, b_jrangle|^2 = 0 $$
        Contradiction.



        So $S = S'$ and hence
        $$ |S| = |S'| = left|bigcup_i in K S_iright| le |K| cdotsup_i |S_i| le |K| cdot aleph_0 = |K| $$
        as desired.







        share|cite|improve this answer














        share|cite|improve this answer



        share|cite|improve this answer








        edited Apr 8 at 11:28









        Matthias Hübner

        876




        876










        answered Nov 7 '12 at 14:27









        martinimartini

        70.9k45992




        70.9k45992



























            draft saved

            draft discarded
















































            Thanks for contributing an answer to Mathematics Stack Exchange!


            • Please be sure to answer the question. Provide details and share your research!

            But avoid


            • Asking for help, clarification, or responding to other answers.

            • Making statements based on opinion; back them up with references or personal experience.

            Use MathJax to format equations. MathJax reference.


            To learn more, see our tips on writing great answers.




            draft saved


            draft discarded














            StackExchange.ready(
            function ()
            StackExchange.openid.initPostLogin('.new-post-login', 'https%3a%2f%2fmath.stackexchange.com%2fquestions%2f232166%2fshowing-the-basis-of-a-hilbert-space-have-the-same-cardinality%23new-answer', 'question_page');

            );

            Post as a guest















            Required, but never shown





















































            Required, but never shown














            Required, but never shown












            Required, but never shown







            Required, but never shown

































            Required, but never shown














            Required, but never shown












            Required, but never shown







            Required, but never shown







            Popular posts from this blog

            Hidroelektrana Sadržaj Povijest | Podjela hidroelektrana | Snaga dobivena u hidroelektranama | Dijelovi hidroelektrane | Uloga hidroelektrana u suvremenom svijetu | Prednosti hidroelektrana | Nedostaci hidroelektrana | Države s najvećom proizvodnjom hidro-električne energije | Deset najvećih hidroelektrana u svijetu | Hidroelektrane u Hrvatskoj | Izvori | Poveznice | Vanjske poveznice | Navigacijski izbornikTechnical Report, Version 2Zajedničkom poslužiteljuHidroelektranaHEP Proizvodnja d.o.o. - Hidroelektrane u Hrvatskoj

            WordPress Information needed

            Oconto (Nebraska) Índice Demografia | Geografia | Localidades na vizinhança | Referências Ligações externas | Menu de navegação41° 8' 29" N 99° 45' 41" O41° 8' 29" N 99° 45' 41" OU.S. Census Bureau. Census 2000 Summary File 1U.S. Census Bureau. Estimativa da população (julho de 2006)U.S. Board on Geographic Names. Topical Gazetteers Populated Places. Gráficos do banco de dados de altitudes dos Estados Unidos da AméricaEstatísticas, mapas e outras informações sobre Oconto em city-data.com